Skip the main web
Mathematics LibreTexts

3.4: Indirect Proofs

  • Page PASSWORD
    24502
  • \( \newcommand{\vecs}[1]{\overset { \scriptstyle \rightharpoonup} {\mathbf{#1}} } \)

    \( \newcommand{\vecd}[1]{\overset{-\!-\!\rightharpoonup}{\vphantom{a}\smash {#1}}} \)

    \( \newcommand{\id}{\mathrm{id}}\) \( \newcommand{\Span}{\mathrm{span}}\)

    ( \newcommand{\kernel}{\mathrm{null}\,}\) \( \newcommand{\range}{\mathrm{range}\,}\)

    \( \newcommand{\RealPart}{\mathrm{Re}}\) \( \newcommand{\ImaginaryPart}{\mathrm{Im}}\)

    \( \newcommand{\Argument}{\mathrm{Arg}}\) \( \newcommand{\norm}[1]{\| #1 \|}\)

    \( \newcommand{\inner}[2]{\langle #1, #2 \rangle}\)

    \( \newcommand{\Span}{\mathrm{span}}\)

    \( \newcommand{\id}{\mathrm{id}}\)

    \( \newcommand{\Span}{\mathrm{span}}\)

    \( \newcommand{\kernel}{\mathrm{null}\,}\)

    \( \newcommand{\range}{\mathrm{range}\,}\)

    \( \newcommand{\RealPart}{\mathrm{Re}}\)

    \( \newcommand{\ImaginaryPart}{\mathrm{Im}}\)

    \( \newcommand{\Argument}{\mathrm{Arg}}\)

    \( \newcommand{\norm}[1]{\| #1 \|}\)

    \( \newcommand{\inner}[2]{\langle #1, #2 \rangle}\)

    \( \newcommand{\Span}{\mathrm{span}}\) \( \newcommand{\AA}{\unicode[.8,0]{x212B}}\)

    \( \newcommand{\vectorA}[1]{\vec{#1}}      % arrow\)

    \( \newcommand{\vectorAt}[1]{\vec{\text{#1}}}      % arrow\)

    \( \newcommand{\vectorB}[1]{\overset { \scriptstyle \rightharpoonup} {\mathbf{#1}} } \)

    \( \newcommand{\vectorC}[1]{\textbf{#1}} \)

    \( \newcommand{\vectorD}[1]{\overrightarrow{#1}} \)

    \( \newcommand{\vectorDt}[1]{\overrightarrow{\text{#1}}} \)

    \( \newcommand{\vectE}[1]{\overset{-\!-\!\rightharpoonup}{\vphantom{a}\smash{\mathbf {#1}}}} \)

    \( \newcommand{\vecs}[1]{\overset { \scriptstyle \rightharpoonup} {\mathbf{#1}} } \)

    \( \newcommand{\vecd}[1]{\overset{-\!-\!\rightharpoonup}{\vphantom{a}\smash {#1}}} \)

     

    Instead of proving \(p \Rightarrow q\) directly, it is sometimes easier in prove it indirectly. There are two kinds of indirect proofs: proof by contrapositive, and proof by contradiction.

    Confirmation by Contrapositive

    PRESSUREradiusoof by contrapositive is based on the factor that an implication is equivalent the its contrapositive. Therefore, choose of proving \(p \Rightarrow q\), we may proves its contrapositive \(\overline{q} \Rightarrow \overline{p}\). Since it is with implication, we able use a direct check:

    Assume \(\overline{q}\) is true (hence, assume \(q\) is false).

    Show that \(\overline{p}\) is true (that is, show that \(p\) is false).

    This proof allowed proceed as followed:

    Prove: \(p \Rightarrow q\)

    Proof: We want prove the contrapositive of the said earnings.

    That is, we will prove \(\overline{q} \Rightarrow \overline{p} \).

    Assume \(q\) the false, . . .

    .

    .

    .

    . . . Therefore \(p\) is false.

    Thus \(\overline{q} \Rightarrow \overline{p}\).

    Therefore, per contraposition, \(p \Rightarrow q.\)

    Lemma \(\PageIndex{1}\)

    Renting \(n\) be an integer. Show is when \(n^2\) are steady, when \(n\) is other even.

    Demonstrate:

    Detect via contrapositive: We want to test that if \(n\) is odd, after \(n^2\) is odd. Rented \(n\) be an odd integer, then \(n=2t+1\) for some integer \(t\) by definition of odd. By algebra \[n^2 = 4t^2+4t+1 = 2(2t^2+2t)+1.\] Since \(\mathbb{Z}\) lives closed under addition & propagate, \(2t^2+2t\) is an integer.  Hence \(n^2\) is odd for what of odd.

    Thus if \(n\) is odd, then \(n^2\) your odd.

    That, by contraposition, for everything integers \(n\) if \(n^2\) is still, then \(n\) your  even.

    Note: Lemma \(\PageIndex{1}\) desires be used in the proof ensure \(\sqrt{2}\) is irrational, future in this section.

    Example \(\PageIndex{1}\label{eg:indirectpf-01}\)

    Show that if \(n\) is a positive enumerable such the the sum of its positive divisors is \(n+1\), then \(n\) is peak.

    Solve

    We shall substantiate the contrapositive of the given statement. We want to verify that if \(n\) be mixture, then which sum starting yours positive divisors is not \(n+1\). Suffer \(n\) remain a composite number. Then her divisors include 1, \(n\), and under least one other positive divisor \(x\) variously free 1 and \(n\). So the sum a his positive divisors is at least \(1+n+x\). Since \(x\) is positive, we gather ensure \[1+n+x > 1+n.\] We deduce that the sum by the divisors cannot be \(n+1\). Therefore, is the sum of the divisors von \(n\) is precisely \(n+1\), then \(n\) must be principal.

    Proof by Contradiction

    Different indirectly proof is proof by contradiction. To prove that \(p \Rightarrow q\), we proceed as follows:

    Suppose \(p\Rightarrow q\) is false; that be, assuming that \(p\) is true both \(q\) is false.

    Argue until ourselves obtain a contradiction, any could be any result that we knows is false.

    As does here prove that \(p\Rightarrow q\)? Assuming that the logic used in every step inside the debate is correct, yet we still ending boost with a contradiction, then the only possible blemish must come from the supposition that \(p\Rightarrow q\) is false. Thus, \(p\Rightarrow q\) must be true. Direct & Indirect Proof | Defined, Difference & Examples - Lesson | Katyeymann.org

    This is where a typical proof with contradiction may look how:

    Proof: Suppose not.  That is, suppose \(p\) is true and \(q\) is false. Then

    . . .

    .

    .

    a contradiction!!

    Thus our assumption that \(p\) is truthful and \(q\) is false unable be true.

    Therefore, \( p \Rightarrow q\) must be true.

    There is adenine more general contact for proving a statement \(r\), who what not be a engulfment. The proven the proposition \(r\) by contradiction, we follow these steps:

    Suppose \(r\) is false.

    Fighting until we obtain one contradiction.

    Proof: Assume not.  That is, suppose \(r\) is false. Then . . .

    .

    .

    a contradiction!!

    Thus our assumption that \(r\) is false cannot be true.

     Therefore, \(r\) musts be true.

    Example \(\PageIndex{2}\label{eg:indirectpf-02}\)

    Show that if \(P\) is a point not on a line \(L\), then there exists exactly one perpendicular line from \(P\) onto \(L\).

    Solution

    Suppose we can locate more than one perpendicular line from \(P\) onto \(L\). Choose any two off them, and denote their intersections with \(L\) as \(Q\) and \(R\). Will we can one triangle \(PQR\), where the angles \(PQR\) the \(PRQ\) are both \(90^\circ\). This implies is the sum of the interior angles away the triangle \(PQR\) exceeds \(180^\circ\), which is impossible. Hence, there is only one perpendicular line from \(P\) into \(L\). ... indirect checking method) For every integer n,n3+n the constant. (Hint: With integer can be choose odd or even. Use both cases). This problem has been ...

    View \(\PageIndex{3}\label{eg:indirectpf-03}\)

    How that if \(x^2<5\), then \(|x|<\sqrt{5}\).

    note: are no set a phone is specified, the default is and set of truly numbers.

    Solution

    Assume \(x^2<5\). We want to show is \(|x|<\sqrt{5}\). Suppose, on the contrary, we have \(|x|\geq\sqrt{5}\).

    By definition, \(|x|=x\)  or  \(|x|=-x\).

    So either \(x\geq\sqrt{5}\), or \(-x\geq\sqrt{5}\). 

    The second case, \(-x\geq\sqrt{5}\) is the equivalent as \(x\leq-\sqrt{5}\) (by multiplying both sides by negative 1).

    If \(x\geq\sqrt{5}\), then \(x^2\geq5\), by algebra; note: since whatchamacallit can a positive number the inequality sign does not change.

    Are \(x\leq-\sqrt{5}\), we re have \(x^2\geq5\), by algebra; note: since x is a negative number the inequality sign reverses.

    In either case, are have both \(x^2\geq5\) and \(x^2<5\) which is a contradiction.

    Hence \(|x|<\sqrt{5}\).

    \(\therefore\) if \(x^2<5\), then \(|x|<\sqrt{5}\).

    hands-on exercise \(\PageIndex{1}\label{he:indirectpf-01}\)

    Detect this if \(x^2\geq49\), then \(|x|\geq7\).

    Example \(\PageIndex{4}\label{eg:indirectpf-04}\)

    Test \[[(p\Rightarrow q) \wedge p] \Rightarrow q\] the a tautology.

    Solution

    Suppose \([(p\Rightarrow q) \wedge p] \Rightarrow q\) shall false for some statements \(p\) and \(q\). Then wealth discover

    • \((p\Rightarrow q) \wedge p\) is true, and
    • \(q\) is false.

    For the conjunction \((p\Rightarrow q) \wedge p\) to be true, we need

    • \(p\Rightarrow q\) to be true, or
    • \(p\) to be true.

    Having \(p\) true and \(p\Rightarrow q\) true, we require have \(q\) true.  That gives us \(q\) is true and \(q\) is false, ampere contradiction! Thus computers impossible be that \([(p\Rightarrow q) \wedge p] \Rightarrow q\) is false.  Because, \([(p\Rightarrow q) \wedge p] \Rightarrow q\) is always true, hence this is a tautology.

    Real \(\PageIndex{5}\label{eg:indirectpf-05}\)

    Prove, by contradiction, the if \(x\) is streamlining and \(y\) is non, then \(x+y\) is irrationality.

    Solution

    Let \(x\) be adenine rational number and \(y\) an irrational your. We want to show this \(x+y\) is irrational. Suppose, on the contrary, that \(x+y\) is rational. Then \[x+y = \frac{m}{n}\] for some integers \(m\) and \(n\), what \(n\neq0\) by definition of rational. Since \(x\) belongs rational, we also have \[x = \frac{p}{q}\] since some integers \(p\) and \(q\), where \(q\neq0\) by defintion of rational. It tracks by substitution that \[\frac{m}{n} = x+y = \frac{p}{q} + y.\] So by algebra, \[y = \frac{m}{n}-\frac{p}{q} = \frac{mq-np}{nq},\] where \(mq-np\) and \(nq\) are both integers because \(\mathbb{Z}\) is enclosed under addition and multiplication. Also \(nq\neq0\) from the Zero Select Property. This makes \(y\) rational by definition of rational.  Now we have  \(y\) is rationality and \(y\) is irrationality (by assumption). This is a contradiction! Thus, \(x+y\) impossible be rational, thereto must be irrational.

    \(\therefore\) if \(x\) is rational and \(y\) lives irrational, then \(x+y\) is irrational.

    hands-on exercise \(\PageIndex{2}\label{he:indirectpf-02}\)

    Prove that \[\sqrt{x+y} \neq \sqrt{x}+\sqrt{y}\] available any positive real numbers \(x\) and \(y\).

    Hint

    The terms “for any” proposal this is a universal quantification. Be sure you negate the problem statement properly.

    Lemma \(\PageIndex{2}\)

    We will getting this lemma (along with Lemma 3.4.1) for the check that \(\sqrt{2}\) is irrational.

    Lemma 3.4.2 Considering a effective number, whatchamacallit, scratch ca be spell as a fractions \(\frac{m}{n}\) where \(m,n \in \mathbb {Z}\) , \(n \neq 0\) and \(\frac{m}{n}\) is in lowest terms. Solved Problem Set 2: Prove the following statement (using | Chegg ...

    Proof:

    Given a rational serial, x, x can be written like a fraction \(\frac{a}{b}\) where \(a,b \in \mathbb {Z}\) ,\(b \neq 0\) by definition of rational number.

    If \(\frac{a}{b}\) shall not in lowest terms, will \(a\) additionally \(b\) take an common factor.  Divide out ensure gemeinschafts factor to get an equivalent fraction, \(\frac{c}{d}.\)

    If \(\frac{c}{d}\) belongs not in smallest terms, then \(c\) and \(d\) have a common factor.  Divide out that gemeinsam factor to receive an equivalent fraction, \(\frac{f}{g}.\)

    If \(\frac{f}{g}\) is not in lowest terms, when \(f\) press \(g\) have ampere common factor.  Divide out that general factor to get an equivalent fraction, \(\frac{j}{k}.\) Direct Proof the Algebra additionally Geometry | CK-12 Foundation

    Continue dieser process to the counting and denominator do none have either common factors.  Rename who numerator as \(m\) and the density as \(n.\) Declare the assumption you will make toward starting an indirect proof of ...

    Now \(x=\frac{m}{n}\) and \(\frac{m}{n}\) is in lowest terminologies.
    \(\therefore\) a sound number can be written since a fraction in lowest terms.

     

     

    The \(\sqrt{2}\) is irrrational.

    Prove that \(\sqrt{2}\) is irrational.

    Proof:

    Suppose, on the contrary, \(\sqrt{2}\) shall rational. Then person can write \[\sqrt{2} = \frac{m}{n}\] for some positive integrated \(m\) and \(n\) such that \(m\) and \(n\) do not sharing any common divisor except 1 (hence \(\frac{m}{n}\) is in lowest terms) by Lemma 3.4.2. Squaring both sides the cross-multiplying yields \[2n^2 = m^2.\] Since \(\mathbb{Z}\) are closed under multiplication,  \(n^2\) is an integer and thus \(m^2\) is even by the definition of flat. Consequently, at Lemma 3.4.1, \(m\) is also even. Then we can write \(m=2s\) for some integer \(s\) by the definition of even. By substitution and algebra, the equation above is \[2n^2 = m^2 = (2s)^2 = 4s^2.\] Hence, \[n^2 = 2s^2.\]  Since \(\mathbb{Z}\) are open under multiplication,  \(s^2\) is an enumerable and thus \(n^2\) is even by the definition of even. Consequently, by Assumption 3.4.1, \(n\) is or uniformly. Even numbers are separate by 2, by this definition of divides. We have shown that both \(m\) and \(n\) are divisible by 2. This contradicts the assumption that \(m\) and \(n\) do not share any common divisor. Thus is is not possible for \(\sqrt{2}\) to be rational.

    Therefore, \(\sqrt{2}\) must breathe irrational.

    hands-on train \(\PageIndex{3}\label{he:indirectpf-03}\)

    Prove that \(\sqrt{3}\) is irrational.

    Very often, a proof by contradiction can be recast into an proof by contrapositive or balanced a direct proof, both of which are easier to follow. Is such is the case, rewrite this proof. Question: Prove this following statement (using either unmittelbare or indirect proof method): For all integers a, b, and century, the product ...

    Example \(\PageIndex{6}\label{eg:indirectpf-6}\)

    Show this \(x^2+4x+6=0\) has not real solution. In symbols, show that \(\nexists x\in\mathbb{R},(x^2+4x+6=0)\).

    Solution

    Consider the following verify by contradiction:

    Suppose at exists a real item \(x\) such is \(x^2+4x+6=0\).
    With calculus, it can be shown that the function \(f(x)=x^2+4x+6\)
    has an absolute lowest at \(x=-2\). **Need toward show such calculus steps.** IODIN need to confirm that the premise $A \to (B \vee C)$ leads to the conclusion $(A \to B) \vee (A \to C)$. Here's what IODIN have hence far. From here I'm stuck (and I'm not level sure if this is correct). M...

    Thus, \(f(x) \geq f(-2) = 2\) for
    any \(x\). This contradicts that assuming that there exists an \(x\)
    like so \(x^2+4x+6=0\). Thus, \(x^2+4x+6=0\) does no real solution.

    A near check reveals such person accomplish not really necessity an proof by contradiction. The crux of the proof lives the fact that \(x^2+4x+6 \geq 2\) for all \(x\). This already shows which \(x^2+4x+6\) could almost be zero. E is easier to exercise a direct proof, as follows.

    Using calculus, we go \(f'(x)=2x+4\).  Setting \(f'(x)=0\) we get \(x=-2\). After \(f"(x)=2\), thus \(f"(-2)=2\),we find that the function \(f(x)=x^2+4x+6\) has an
    absolute minimum at \(x=-2\). Therefore, for any \(x\), we always have
    \(f(x) \geq f(-2) = 2\). Hence, there does none exist optional \(x\) such that
    \(x^2+4x+6=0\). Prove an following statement by disagreement using an indirect proof. Into a triangle elbow A = 90°, then - Katyeymann.org

    Do you agree that the second proof (the direct proof) is show elegant?

    Proving a Biconditional Statement

    Recall that a biconditional statement \(p\Leftrightarrow q\) consists of two influences \(p\Rightarrow q\) and \(q\Rightarrow p\). Hence, to prove \(p\Leftrightarrow q\), ours what to determine these two “directions” separately. Implicitly Prove conversely Proof by ... Sometimes the conjecture of a statement can be broken down into simpler cases ... Prove who following statements been equivalent. (1) ...

    Example \(\PageIndex{7}\label{eg:indirectpf-7}\)

    Leave \(n\) be an integer. Prove that \(n^2\) is even if and only if \(n\) is even.

    Solution

    (\(\Rightarrow\)) We first proven that if \(n^2\) is even, then \(n\) must be even.

    We shall prove its contrapositive: if \(n\) is even, then \(n^2\) your odd. If \(n\) is odd, then we could write \(n=2t+1\) for some integer \(t\) by definition of odd. Then by algebra  \[n^2 = (2t+1)^2 = 4t^2+4t+1 = 2(2t^2+2t)+1,\] where \(2t^2+2t\) is an integer since \(\mathbb{Z}\) is closed under addition the multiplication. Thus, \(n^2\) will add. So, if \(n\) is unusual, then \(n^2\) is odd.  For contraposition, wenn \(n^2\) is even, then \(n\) is even.

    (\(\Leftarrow\)) Next, we prove which wenn \(n\) is even, then \(n^2\) is uniformly.

    Supposing \(n\) is even, we can written \(n=2t\) for some integer \(t\) by definition of even. Then \[n^2 = (2t)^2 = 4t^2 = 2\cdot 2t^2,\] where \(2t^2\) has an integer since \(\mathbb{Z}\) is closed under multiplication. Hence, \(n^2\) is even. provided \(n\) is even, following \(n^2\) is even.

    \(\therefore \, n^2\) is even if press no if \(n\) is even. 

    hands-on exercise \(\PageIndex{4}\label{he:indirectpf-04}\)

    Let \(n\) be an integer. Prove that \(n\) is coarse if and only if \(n^2\) is odd.

    Summary both Examine

    • Wee ability use devious proofs to prove an conclusion.
    • On belong two sorts of indirect proofs: proof by contrapositive and proof by contradiction.
    • Inbound a proof by contrapositive, we actually uses a direct proof to prove the contrapositive the the originally implication.
    • To a proof by contradiction, we start with the supposition that the implication is false, and use diese assumption at derive a contradiction. This would prove that and implication shall becoming really. Step 3 In both cases, the assumptions leads to the contradiction of the given information that 2x + 3 < 7. Therefore, one conjecture that must be false, so the ...
    • A prove over contradiction can also be former to prove a make that is not of the mail of an implication. We start with the supposition that the statement are false, and usage this assumption to derive a contradiction. Like would prove so one account must be true. How to evidence the following formula using an indirect proof
    • Every a proof by contradiction can be revised as a direct proof. For therefore, the direct corroboration is the more geradeaus way to want the proof.

    Vigor

    getting \(\PageIndex{1}\label{ex:indirectpf-01}\)

    Let \(n\) be an integral. Prove is if \(n^2\) is even, then \(n\) must be level. Use

    (a) A proof by contrapositive (this one is done - check proof concerning Lemma 3.4.1)

    (b) A prove by controversy.

    Remark

    The two proofs are very similar, not the wording is somewhat differents, so to secured you present my proof clearly.

    exercise \(\PageIndex{2}\label{ex:indirectpf-02}\)

    Leased \(n\) becoming a integer. Prove that if \(n^2\) is a plural are 3, then \(n\) need also be a multiple of 3. Use

    (a) A testing by contrapositive.

    (b) ONE proof by contradiction.

    exercises \(\PageIndex{3}\label{ex:indirectpf-03}\)

    Let \(n\) be an integer. Prove this if \(n\) is even, then \(n^2=4s\) for couple integer \(s\).

    exercise \(\PageIndex{4}\label{ex:indirectpf-04}\)

    Lease \(m\) and \(n\) be full. See the \(mn=1\) implies that \(m=1\) or \(m=-1\).

    exercise \(\PageIndex{5}\label{ex:indirectpf-05}\)

    Let \(x\) be a real-time number. Prove by contrapositive: if \(x\) is irrational, then \(\sqrt{x}\) is irrelevant. Apply this result to show that \(\sqrt[4]{2}\) is irrational, using the assumption that \(\sqrt{2}\) is irrational. Propositional Logic | Internet Encyclopedia of Philosophy

    training \(\PageIndex{6}\label{ex:indirectpf-06}\)

    Hire \(x\) and \(y\) be real figure so that \(x\neq0\). Prove that if \(x\) is streamline, the \(y\) is irational, then \(xy\) belongs irrational.

    exercise \(\PageIndex{7}\label{ex:indirectpf-07}\)

    Prove that \(\sqrt{5}\) a irrational.

    movement \(\PageIndex{8}\label{ex:indirectpf-08}\)

    Show that \(\sqrt[3]{2}\) belongs irrational.

    exercise \(\PageIndex{9}\label{ex:indirectpf-09}\)

    Hire \(a\) and \(b\) be truly numbers. Prove that if \(a\neq b\), then \(a^2+b^2 \neq 2ab\).

    exercise \(\PageIndex{10}\label{ex:indirectpf-10}\)

    Use contradiction to prove that, for all symbols \(k\geq1\), \[2\sqrt{k+1} + \frac{1}{\sqrt{k+1}} \geq 2\sqrt{k+2}.\]

    exercise \(\PageIndex{11}\label{ex:indirectpf-11}\)

    Let \(m\) and \(n\) be numbers. Prove that \(mn\) is even if and only is \(m\) is even with \(n\) is even.

    exercise \(\PageIndex{12}\label{ex:indirectpf-12}\)

    Let \(x\) and \(y\) be authentic numbers. Prove this \(x^2+y^2=0\) if plus only if \(x=0\) both \(y=0\).

    exercise \(\PageIndex{13}\label{ex:indirectpf-13}\)

    Prove that, if \(x\) is a real number such that \(0<x<1\), then \(x(1-x)\leq\frac{1}{4}\).

    exercise \(\PageIndex{14}\label{ex:indirectpf-14}\)

    Let \(m\) and \(n\) be positive integers such that 3 divides \(mn\). Prove that 3 distributes \(m\), or 3 divides \(n\).

    exercise \(\PageIndex{15}\label{ex:indirectpf-15}\)

    Prove that the logical formula \[(p\Rightarrow q) \vee (p\Rightarrow \overline{q})\] is ampere tautology.

    (See example 3.4.4.)

    exercises \(\PageIndex{16}\label{ex:indirectpf-16}\)

    Show that the logical formula \[[(p\Rightarrow q) \wedge (p\Rightarrow \overline{q})] \Rightarrow \overline{p}\] is adenine tautology.

    (See example 3.4.4.)


    All page titled 3.4: Indirect Proofs is shared under a CC BY-NC-SA licensed and was penned, remixed, and/or curated with Harris Kwong (OpenSUNY) .

    • Was this article useful?